LSAT and Law School Admissions Forum

Get expert LSAT preparation and law school admissions advice from PowerScore Test Preparation.

 rileeh
  • Posts: 2
  • Joined: Jun 11, 2020
|
#76141
why is the contrapositive of rule 2 not : no F1 and no T1 ---> no F and no T
 Paul Marsh
PowerScore Staff
  • PowerScore Staff
  • Posts: 290
  • Joined: Oct 15, 2019
|
#76214
Hi rileeh! What you wrote is the correct contrapositive of rule 2; you're right on the money! If the first slot is not occupied by F or T, then the newspaper does not contain F or T features. Nice job.
User avatar
 ddddd8888899999
  • Posts: 11
  • Joined: Nov 04, 2021
|
#92185
Hello,

Is it theoretically possible to have one feature M spanning the first two slots, a second feature M on the third slot, feature I on 4, and then feature M on 5.

In short, can we have two different features of the same letter consecutively ? It seems that the rules do not preclude this, but then it would be difficult to know if, in the example above, the first three slots are occupied by one 3 slotted feature M, or two features M, one of 2 slots and the second of 1 slot.


Thanks
 Adam Tyson
PowerScore Staff
  • PowerScore Staff
  • Posts: 5153
  • Joined: Apr 14, 2011
|
#92186
It is indeed possible, ddddd8888899999, and see my explanation earlier in this thread for a similar arrangement. As to telling apart a single variable that is stretched across multiple spaces vs multiple of the same type in consecutive spaces, I chose to do things like this:

( _M_ ) M I M

See how the first M is spanning across two blank spaces? Putting it in parentheses helps me further to see that it's just one variable covering those two spaces.

If I want to show two consecutive Ms that are not the same feature, I would do it without the parentheses and with a letter in each slot, like so:

M M M I M
User avatar
 Adam354
  • Posts: 29
  • Joined: Feb 08, 2022
|
#93743
"One in 2,3, or 4 must occur, so even if we take the wrong interpretation, it doesn't matter, we still get it right."

The rule states that if a feature occupies more than one slot, it must occupy consecutive slots.

Oh, I see it now. It was a trick. Say there are 3 M's.

The rule must be fulfilled. How many M's do we need to fill that rule? 2.

Once 2 M's are used, the rule is fulfilled, because it is now occupying consecutive slots.

It does not state that ALL M's must be in consecutive slots.

I would say it is unreasonably misleading, however, there is an answer that fulfills BOTH conditions.

One in 2,3, or 4 must occur, so even if we take the wrong interpretation, it doesn't matter, we still get it right, and perhaps the potentially misleading interpretation, was only given as a possible answer in order to distract the test taker. Worst case scenario, you had to quickly scroll back through the previous two questions to make sure this did not affect the other questions.
 Adam Tyson
PowerScore Staff
  • PowerScore Staff
  • Posts: 5153
  • Joined: Apr 14, 2011
|
#93750
What about this game did you find unreasonably misleading, Adam? It's confusing and challenging, certainly, but in my view everything about it is reasonable and fair (and whatever else we may think of LSAC and the pains they inflict upon us, I think the test content is consistently reasonable and fair). This game repeatedly tests our understanding of what the rules require and also of what they do not require. In other words, it tests whether students have made unwarranted inferences about how the variables may be placed in the base. In that regard it reminds me a lot of the Zones/Subzones game from PT 67, which I highly recommend everyone study to get the importance of careful reading and of not making assumptions.

Get the most out of your LSAT Prep Plus subscription.

Analyze and track your performance with our Testing and Analytics Package.